If the production rate is 975 brownies per hour at a bakery, how many brownies will be produced in an 8 hour shift?

Answers

Answer 1

Answer:

7800 brownies will be produced in an 8 hour shift

Step-by-step explanation:

Given : The production rate is 975 brownies per hour at a bakery

To Find :  how many brownies will be produced in an 8 hour shift?

Solution:

No. of brownies produced in 1 hour = 975

We are supposed to find No. of brownies produced in 8 hour

No. of brownies produced in 8 hour =  975 x 8=7800

Hence 7800 brownies will be produced in an 8 hour shift

Hope this helps ФωФ


Related Questions

Is f(x) =(x+5)2 a function, an odd function, both or neither

Answers

Answer: Neither even, nor odd

Explanation:

f(x) = (x+5)^2 = x^2+10x+25 = x^2+10x^1+25x^0

The exponents for that last expression are 2, 1, 0

The mix of even and odd exponents in the standard form means f(x) is neither even nor odd. We would need to have all exponents even to have f(x) even, or have all exponents odd to have f(x) be odd.

Julio wants to solve the system shown using the elimination method. Which is the best way to begin?
(x - 12y = 2
-4x + 7y = 12
Add the equations
b. Multiply each term in x - 12y = 2 by 4 and add it to the other original equation.
This system of equations has no solution, so Julio should not do anything.
d. Multiply each term in x - 12y = 2 by 4 and add it to the other original equation.
c.

Answers

Answer:

B. Multiply each term in x - 12y = 2 by 4 and add it to the other original equation.

Step-by-step explanation:

The expression are two linear equation and can be solved simultaneously

[tex]x - 12y = 2------------------1[/tex]

[tex]-4x + 7y = 12----------------------2[/tex]

1. we need to multiply each term in eqn 1 by 4 and add it to the other

original equation(2).

[tex]4x - 48y = 8----------------3\\-4x + 7y = 12---------------2\\\\[/tex]

Adding both 3 and 2 we have

[tex]4x - 48y = 8----------------3\\-4x + 7y = 12---------------2\\\\\\[/tex]

2. once we have gotten the value of y

we then substitute it in any of the equations to solve for x

This bag has an unknown number of lollipops and jelly snakes in it. I know that Pr (jelly snake) = 4/5. How many jelly snakes could I have in the bag, if I know I have over 10 lollies total?

Answers

Answer:

40.

Step-by-step explanation:

Let J represent jelly snake.

Let L represent lollipops

Let S represent sample space

Let y represent the number of jelly snake in the bag. I.e

Number of J = y

Number of L = 10

Number of S = 10 + y

From the question given, we were told that the probability of jelly snake P(J) is 4/5.

But probability of jelly snake, P(J) is given by:

P(J) = nJ/nS

nJ = y

nS = 10 + y

P(J) = 4/5

Thu, we can obtain the number jelly snake, y, in the bag as follow:

P(J) = nJ/nS

4/5 = y/(10 + y)

Cross multiply

5y = 4(10 + y)

Clear bracket

5y = 40 + 4y

Collect like terms

5y – 4y = 40

y = 40

Therefore, the number of jelly snake in the bag is 40

On a coordinate plane, a triangle has points (negative 5, 1), (2, 1), (2, negative 1).
Use the drop downs to answer the following questions about the distance between the points (−5, 1) and (2, −1).

What is the distance of the horizontal leg?

What is the distance of the vertical leg?

Use the Pythagorean theorem. What is the distance between the two points?

Answers

Answer:

The answer is below

Step-by-step explanation:

The points of the triangle are  (- 5, 1), (2, 1), (2, - 1). The distance between two points is given by:

[tex]Distance=\sqrt{(x_2-x_1)^2+(y_2-y_1)^2}[/tex]

The horizontal leg is formed by points with the same y axis. Therefore the points that make up the horizontal leg is (- 5, 1), (2, 1). The Distance of the horizontal leg is:

[tex]Horizontal\ leg=\sqrt{(2-(-5))^2+(1-1)^2}=\sqrt{7^2+0}=7\ units[/tex]

The vertical leg is formed by points with the same x axis. Therefore the points that make up the vertical leg is (2 1), (2, 1-). The Distance of the vertical leg is:

[tex]Vertical\ leg=\sqrt{(2-2)^2+(-1-1)^2}=\sqrt{0+(-2)^2}=2\ units[/tex]

The hypotenuse is gotten using Pythagorean theorem. It is gotten by:

Hypotenuse² = (Horizontal leg)² + (Vertical leg)²

Hypotenuse² = 7² + 2²

Hypotenuse² = 49 + 4 = 53

Hypotenuse = √53

Hypotenuse = 7.28 unit

Answer:

The answer are 7, 2 and 53

Step-by-step explanation:

Will Give Brainliest, Answer ASAP m∠O =
m∠N =

Answers

Answer:

∠ O = 61°, ∠ N = 119°

Step-by-step explanation:

In a parallelogram

Consecutive angles are supplementary

Opposite angles are congruent, thus

x + 2x - 3 = 180

3x - 3 = 180 ( add 3 to both sides )

3x = 183 ( divide both sides by 3 )

x = 61°

Thus

∠ O = ∠ M = x = 61°

∠ N = ∠ P = 2x - 3 = 2(61) - 3 = 122 - 3 = 119°

The amounts of time per workout an athlete uses a stairclimber are normally​ distributed, with a mean of minutes and a standard deviation of minutes. Find the probability that a randomly selected athlete uses a stairclimber for​ (a) less than ​minutes, (b) between and ​minutes, and​ (c) more than minutes. ​(a) The probability that a randomly selected athlete uses a stairclimber for less than minutes is nothing. ​(Round to four decimal places as​ needed.) ​(b) The probability that a randomly selected athlete uses a stairclimber between and minutes is nothing. ​(Round to four decimal places as​ needed.) ​(c) The probability that a randomly selected athlete uses a stairclimber for more than minutes is nothing.

Answers

Answer:

Step-by-step explanation:

Let S be the sample space, n(S) = 60

a) Let A be the event that the selected athlete uses

s less than a minute, n(A) = 59

The probability that a randomly selected athlete uses less a minute,  P(A) = n(A)/n(S) = 59/60 = 0.9833

b) 1 - 0.9833 = 0.0167

c)  1 - 1 = 0

Puzzle corner
Look Before You Leap!
See how long it takes you to work out the
following:
(1 x2)×(3 x 4)×(586)×(7 x 8) x (
9×0)

Answers

Answer:

0

Step-by-step explanation:

Notice that the last factor is null (9×0)

So the result will be null since any number that is multiplied by 0 equals 0.

Dawn and Jackson have baseball cards in the ratio of 2:3. Together, they have a total of 60 baseball cards. How many baseball cards does each child have?

Answers

Answer:

24 and 36

Step-by-step explanation:

2x + 3x = 60

5x = 60

x = 12

Dawn has 2(12) = 24

Jackson has 3(12) = 36

Step-by-step explanation:

To find the number of baseball cards each person received we must first find the total parts

That's

2 + 3 = 5

For Dawn

Dawn's part is 2

We have

2/5 × 60

= 24 baseball cards

For Jackson

Jackson's part is 3

That's

3/5 × 60

= 36 baseball cards

Hope this helps you

Which number line represents the solution set for the inequality –negative StartFraction one-half EndFraction x is greater than or equal to 4.x ≥ 4?

A number line from negative 10 to 10 in increments of 2. A point is at negative 2 and a bold line starts at negative 2 and is pointing to the left.
A number line from negative 10 to 10 in increments of 2. A point is at negative 8 and a bold line starts at negative 8 and is pointing to the left.
A number line from negative 10 to 10 in increments of 2. A point is at negative 2 and a bold line starts at negative 2 and is pointing to the right.
A number line from negative 10 to 10 in increments of 2. A point is at negative 8 and a bold line starts at negative 8 and is pointing to the right.

Answers

Answer:

it's b :)

Step-by-step explanation:

A number line which represents the solution set for the given inequality is: option B.

What is a number line?

A number line refers to a type of graph with a graduated straight line which contains numerical values (both positive and negative numbers) that are placed at equal intervals along its length.

Next, we would solve the given inequality:

-½x ≥ 4

-x ≥ 4 × 2

x ≤ -8.

Therefore, a number line which represents the solution set for the given inequality is a number line from -10 to 10 in increments of 2 with a point at -8 and a bold line starts at -8 while pointing to the left.

Read more on number line here: brainly.com/question/24644930

#SPJ9

PLS HELP I REALLY NEED IT

Answers

Answer:

x=9

Step-by-step explanation:

<B = <E  from the concurrency statement

5x = 45

Divide by 5

5x/5 = 45/5

x = 9

Answer:

Hey there!

These are similar triangles, and similar triangles have congruent angles.

Thus, we have 5x=45

Simplifying, we have x=9

Let me know if this helps :)

Brian invested his savings in two investment funds. The $8000 that he invested in Fund A returned a 4% profit. The amount that he invested in Fund B returned a 1% profit. How much did he invest in Fund B, if both funds together returned a 2% profit?

Answers

Answer: Brian invested $16000 in Fund B .

Step-by-step explanation:

Let x be the amount Brian invested in Fund B.

Given, The $8000 that he invested in Fund A returned a 4% profit. The amount that he invested in Fund B returned a 1% profit.

i.e. profit on Fund A = 4% of 8000 = 0.04 ×8000 = $320

Profit on Fund B = 1% of x = 0.01x

Together they earn 1% profit, i.e. Combined profit = 2% of (8000+x)

= 0.02(8000+x)

As per question,

Combined profit=Profit on Fund A+Profit on Fund B

[tex]\Rightarrow\ 0.02(8000+x) =320+0.01x\\\\\Rightarrow\ 0.02(8000) +0.02x=320+0.01x\\\\\Rightarrow\ 160+0.02x=320+0.01x\\\\\Rightarrow\ 0.02x-0.01x=320-160\\\\\Rightarrow\ 0.01x=160\\\\\Rightarrow\ x=\dfrac{160}{0.01}\\\\\Rightarrow\ x=16000[/tex]

Hence, Brian invested $16000 in Fund B .

Find the radius of a circle that has an area of 6.76 cm². Use it for pi.

Answers

Answer:

radius = 1.47 cm

Step-by-step explanation:

Area of a circle = πr²

where

r is the radius

From the question

Area = 6.76 cm²

To find the radius substitute the value of the area into the above formula and solve for the radius

That's

[tex]6.76 = \pi \: {r}^{2} [/tex]

Divide both sides by π

We have

[tex] {r}^{2} = \frac{6.76}{\pi} \\ r = \sqrt{ \frac{6.76}{\pi} } [/tex]

r = 1.46689291

We have the final answer as

radius = 1.47 cm

Hope this helps you

WILL MARK BRAINLIEST FOR CORRECT ANSWER. <3 <3 The area of the following rectangle is 24 square units. A. Write an equation that can be used to find the value of n. B. Solve the equation to find the value of n. In your answer, show all of your work.

Answers

Because area is multiplication lets multiply the sides and make an equation:

A 2(n-3) = 24

n-3=(12)

B n= 15

15-3 = 12

12 x 2 = 24

Answer:

2(n-3)=24

n=15

Step-by-step explanation:

2(n-3)=24

2n-6=24

2n=30

n=15

Given f(x) = 2x - 7, complete parts (a) through (c).
A. Solve f(x)=0.
B. What do the answers to parts (a) and (b) tell you about the graph of y=f(x)

Answers

Answer:

a) x=7/2

Step-by-step explanation:

a) since f(x) is=0, plug in 0 to → f(x)=2x-7 [this f(x)]. you would get 0=2x-7. solve for x by adding 7 and dividing by 2 which you get x=7/2.

Then value of [tex]x[/tex] is 7/2

What is function?

Functions are the fundamental part of the calculus in mathematics. The functions are the special types of relations. A function in math is visualized as a rule, which gives a unique output for every input . Mapping or transformation is used to denote a function in math. These functions are usually denoted by letters. The domain is defined as the set of all the values that the function can input while it can be defined. The range is all the values that come out as the output of the function involved. Co-domain is the set of values that have the potential of coming out as outputs of a function.

given function:

[tex]f(x)[/tex]= 2[tex]x[/tex] -7

So,[tex]f(x)[/tex]= 0

2[tex]x[/tex] -7=0

2[tex]x[/tex]= 7

[tex]x[/tex]= 7/2

The graph is attached below.

Learn more about function here:

https://brainly.com/question/12431044

#SPJ2

Find the square root of 2601 by prime factorization (USE MULTIPLICATION METHOD TO SOLVE THE ABOVE QUESTION)

Answers

2601|3

867|3

289|17

17|17

1

[tex]\sqrt{2601}=\sqrt{3^2\cdot17^2}=3\cdot17=51[/tex]

Let u = , v = . Find u + v. (1 point)

Answers

Answer:

i couldnt find a solution to the equation all i could get is u +v

Step-by-step explanation:

what were you trying to say when you said "let u="

Answer:

4.8

Step-by-step explanation:

(-3,4) + (8,2) = 4.8

The perimeter of an isosceles triangle is 32 inches. If the base is longer than half of the two other equal sides by 2 inches, find the length of all sides of this triangle.

Write as a equation.​

Answers

Answer:

Step-by-step explanation:

Let equal sides of an isosceles triangle = a inches

Base = [tex]\frac{1}{2}a+2[/tex] inches

Perimeter  = 32 inches

a + a + [tex]\frac{1}{2}a+2[/tex] = 32

[tex]2a + \frac{1}{2}a+2 = 32\\\\\frac{2a*2}{1*2}+\frac{1}{2}a+2=32\\\\\frac{4a}{2}+\frac{1}{2}a+2=32\\\\\frac{5}{2}a+2 = 32\\\\[/tex]

Subtract 2 from both sides

[tex]\frac{5}{2}a=32-2\\\\\frac{5}{2}a=30\\\\a=30*\frac{2}{5}\\\\a=6*2[/tex]

a = 12 inches

base = [tex]\frac{1}{2}*12+2[/tex]

        = 6 + 2

Base = 8 inches

find 5 rational numbers between -3 and -4

Answers

Answer:

-3.2, -3.4, -3.6, -3.8, -3.9

Step-by-step explanation:

Hey there!

Well rational numbers can be a decimal as long as it can be turned into a fraction, meaning 3.5 is a rational number.

So rational numbers between -3 and -4 are,

-3.2, -3.4, -3.6, -3.8, -3.9

Hope this helps :)

For example:

-3.5

-3.0040012

-3.(91)

-3.70(77)

-15/4

PLEASE HELP ME WORTH 20 POINTS It looks like the graph of the parents function f(x)x^2. However:
- It has been reflected (flipped) over the x-axis
-It has been shifted down 4 units.
-It had been shifted left 1 unit

Step 1: Start with the equation f(x) = x2. Write the equation for the graph of g(x) that has been reflected, or flipped, over the x-axis.

Step 2: Use the equation you wrote in Step 1. Write the equation for the graph of g(x) that has also been shifted down 4 units.

Step 3: Use the equation you wrote in Step 2. Write the equation for the graph of g(x) that has also been shifted left 1 unit.

Answers

flipped : [tex]-x^2[/tex]

moving down: [tex] -x^2+4[/tex]

shifting left [tex] -(x+1)^2+4[/tex]

expanding it: [tex] -x^2-2x+3[/tex]

Answer:

1. f(x)=x^2

f(x)=-x^2

2. f(x)=-x^2-4

3. f(x)=-(x+1)^2-4

draw the graph of linear equation 5y = 3x + 18 on a cartesian plane. From the graph check weather (-2,4) is the solution of the linear equation or not PLS URGENT ANSWER

Answers

Answer:

The point (-2, 4) is not a solution of the linear equation, 5·y =  3·x + 18

Please find attached the required graph of the linear equation 5·y =  3·x + 18 written in the form y = 3/5·x + 18/5

Step-by-step explanation:

The given equation is 5·y = 3·x + 18, from which we have;

y = 3/5·x + 18/5

To draw the graph, we generate for vales of y corresponding to values of x as follows;

x,      y

-6,    0

-5,    0.6

-4,    1.2

-3,    1.8

-2,    2.4

-1,      3

0,      3.6

1,       4.2

2,      4.8

3,      5.4

4,       6

5,       6.6

6,       7.2

7,        7.8

8,        8.4

9,        9

10,      9.6

11,       10.2

12,      10.8

13,      11.4

14,       12

15,      12.6

16,      13.2

Therefore, when y = 0, x = -6, when x = 0, y = 3.6, when x = -2, y = 2.4, when y = 4, x = -2, x = 6

Therefore, the point (-2, 4) is not a solution of the linear equation, 5·y =  3·x + 18

The sum of three consecutive numbers is greater than 40. The inequality that represents this is x+x+1+x+2>40. Which value of x hold true for the inequality?

Answers

Answer:

x can be any integer greater than 12.

Step-by-step explanation:

x + x + 1 + x + 2 > 40

3x + 3 > 40

3x > 37

x > 12 1/3

x can be any integer greater than 12.

761.8 x 10^-8 Express the number in scientific notation. A) 7.618 x 10^-6 B) 7.618 x 10^-8 C) 7.618 x 10^2 D) 7.618 x 10^6

Answers

Answer:

[tex]\huge\boxed{A)\ 7.618\times10^{-6}}[/tex]

Step-by-step explanation:

The scientific notation:

[tex]a\cdot10^n[/tex]

where

[tex]1\leq a<10;\ n\in\mathbb{Z}[/tex]

We have

[tex]761.8\times10^{-8}[/tex]

We need to move the decimal point two places to the left.

[tex]\underbrace{(7.618\times10^2)}_{=761.8}\times10^{-8}=7.618\times(10^2\times10^{-8})[/tex]

use

[tex]a^n\cdot a^m=a^{n+m}[/tex]

[tex]=7.618\times10^{2+(-8)}=7.618\times10^{-6}[/tex]

Answer:

a

Step-by-step explanation:

type the correct answer in the box. use numerals instead of words. what value of x makes this equation true? x/6 - 7 = -4

Answers

[tex]\dfrac{x}{6}-7=-4\\\dfrac{x}{6}=3\\x=18[/tex]

Answer:

x = 18

Step-by-step explanation:

x/6 - 7 = -4

Add 7 to each side

x/6 - 7+7 = -4+7

x/6 = 3

Multiply each side by 6

x/6 *6 = 3*6

x = 18

On a coordinate plane, a piecewise function has 3 lines. The first line has an open circle at (negative 9, negative 2), continues horizontally at y = negative 2, then has an open circle at (0, negative 2). The second line has an open circle at (0, 1), continues up with a positive slope, then has an open circle at (4, 9). The third line has an open circle at (4, negative 2), continues down with a negative slope, then has an open circle at (8, negative 4).
What is the domain indicated on the graph for each

Answers

Answer:  D: x = (-9, 0) U (0, 4) U (4, 8)

Step-by-step explanation:

Line 1:      y = -2               where      -9 < x < 0

Line 2:     y = 2)x + 1        where       0 < x < 4

Line 3:    y = -(1/2)x + 6    where       4 < x < 8

Domain represents the x-values.  Since all of them are open dots, the intervals are strictly less than (<).

-9 < x < 0   and   0 < x < 4   and   4 < x < 8 is the union of these intervals

-9 < x < 0   U   0 < x < 4   U   4 < x < 8

Interval Notation: D: x = (-9, 0) U (0, 4) U (4, 8)

Answer:

1st piece:  

✔ –10 < x < 0

2nd piece:  

✔ 0 < x < 4

3rd piece:  

✔ 4 < x < 8

Step-by-step explanation:

The side of an Equileteral triangle is 12cm. What is its Area?

Answers

Answer:

A = 62.35 cm²

Step-by-step explanation:

Use the area formula A = [tex]\frac{\sqrt{3}a^2}{4}[/tex], where a is the side length.

Plug in the values:

A = [tex]\frac{\sqrt{3}(12^2)}{4}[/tex]

A = [tex]\frac{\sqrt{3}(144)}{4}[/tex]

A = 62.35 cm²

Maria has eight black marbles, fourteen clear marbles, and twelve blue marbles in a bag. If she picks two marbles at random, without replacement, what is the probability that she will select a blue marble first, then a clear marble?

Answers

Answer:

[tex]\boxed{0.15}[/tex]

Step-by-step explanation:

Part 1: Solve for the total amount of marbles

To solve for the probability of certain events, a population is needed to derive this information from. In order to find this population, add up the amounts of each marble.

8 + 14 + 12 = 34 marbles

Part 2: Determine the probabilities

Now, given the amounts of marbles, simply multiply the ratios of blue marbles to total marbles and the ratio of clear marbles to total marbles to get the combined probability.

[tex]\frac{12}{34}*\frac{14}{33} = \frac{28}{187} \approxeq 0.1497 \approxeq 0.15 * 100 = 15[/tex]

The probability of these events occurring simultaneously is 15%.

PLS HELP. i really need this fast ill give brainliest too

Answers

Answer:

24 square units

Step-by-step explanation:

Use the formula for area of a parallelogram to solve.  The base is 6 units, and the height is 4 units.

A = bh

A = (6)(4)

A = 24 square units

The area of the parallelogram is 24 square units.

The numbers in the select boxes are 4 7 and 3 I put them with the photos. Can someone help

Answers

Answer:

4:7

Step-by-step explanation:

4 grape candies : 7 total candies (grape + cherry)

Answer:

It is simply 4:3

can u help me with this?​

Answers

Answer:  Yes. The sales tax is 5% which equals $4.20 for $84

Step-by-step explanation:

[tex]\dfrac{0.60}{12}=0.05\qquad \rightarrow 5\%\\\\\\\dfrac{1.20}{24}=0.05\qquad \rightarrow 5\%\\\\\\\dfrac{1.80}{36}=0.05\qquad \rightarrow 5\%\\\\\\\dfrac{2.40}{48}=0.05\qquad \rightarrow 5\%[/tex]

The sales tax rate is proportional for the values in the table.

$84 x 0.05 = $4.20

The sales tax on a purchase of $84 is $4.20

A store sold 50 copies of a magazine for $150. Each copy of the magazine costs the same. Which equation and set of ordered pairs best represents the price, in dollars, of a certain number of copies of the magazine? (1 point) Select one: a. Y = 3x; (1, 3), (2, 6), (3, 9) b. Y = 4x; (1, 4), (2, 8), (3, 12) c. Y = 5x; (1, 5), (2, 10), (3, 15) d. Y = 6x; (1, 6), (2, 12), (3, 18) Plz answer quick!

Answers

Answer:

Option a. Y=3x

Step-by-step explanation:

Let us use cross multiplication method.

Let the cost of 1 magazine be x.

No. of copies                               Cost

1)50                                                $150

2)1                                                      x

50x=150 x 1               equation(1)

x=150/50

x=$3

Now see equation (1),

150=50x

150=50 x 3

Here let us represent the cost as y and no. of copies as x.

Y=3x

Therefore, a. Y=3x is the right answer.

Thank you!

                                             

Other Questions
Volume 1 (3)3 = 367SSCE/JME-TYPE OF2The area of an equilateral triangle of side 8 cm isA. 16V3 cm? B. 32/3 cmB.48 cmcm?D.36V3 cmAparallelogramof area 425 cmhas a height o The dosage for a certain drug calls for 20mg per kg per day and is divided into two doses(1every 12 hours) if a person weighs 197 pounds how much of the drug should be given each dose =Graphing an integer function and finding its range for a given...The function h is defined as follows for the domain given.h(x) = 2 -2x, domain = {-3, -2, 1, 5}Write the range of h using set notation. Then graph h. The application layer process that sends mail uses __________. When a client sends email, the client process connects with a server process on well-known port __________. A client retrieves email, however, using one of two application layer protocols: ________ or ________. With ________, mail is downloaded from the server to the client and then deleted on the server. The server starts the __________ service by passively listening on TCP port __________ for client connection requests. However, when a client connects to ta server running __________, copies of the messages are downloaded to the client applications. The original messages are kept on the server until they are manually deleted. The smartest thing a firm involved in an oligopoly market could do is to cut their prices and capture more of the market share from their competitors. a) We learned in class that the best move would be to raise prices. b) We also learned that cutting prices on an elastic demand curve will be a smart way of getting more revenues. c) Cutting prices is no gaurantee of success. Indeed if the firm does capture more market share and customers, then their costs will go up and it will be harder for them because they will have lower profit margins - if they can earn any profit at all. d) Both A and C are correct. opposite meaning of old Janis owns and operates a store in a country experiencing a high rate of inflation. In order to prevent the value of money in her cash register from falling too quickly, Janis sends an employee to the bank four times per day to make deposits in a interest-bearing account that protects the store's revenues from the effects of inflation.(a) This is an example of the:i. menu costsii. unit of account costsiii. shoe leather costs of inflation.(b) Explain briefly the nature of the costs imposed. the amount of gas in sarahs car is uniformly distributed between 1 and 16 gallons. Calculate the probability that the amount of gas is exactly 7 gallons Decide all proper subsets of A { 8 ,7 ,6 ,5 ,4 ,3 ,2 ,1} = A 1- { 4 ,3 ,2 ,1} 2- { } 3- { 9 ,8 ,7 } 4- { 11 ,2} 5- { 5 } If the price that determined where marginal revenue equaled marginal cost were below the bottom of the average variable cost curve, then the profit-maximizing, monopolistically competitive firm would In your research for your Egyptian and Mayan compare and contrast essay, you discovered various works of art created by these cultures. What stood out for you when you studied them? Would this type of art appeal to you for your own personal use? Why do you think this art was important to them? Find the (a) mean, (b) median, (c) mode, and (d) midrange for the data and then (e) answer the given question. Listed below are the weights in pounds of 1111 players randomly selected from the roster of a championship sports team. Are the results likely to be representative of all players in that sport's league? 278303186292 276 205208236278 198 208 a. Find the mean.The mean is ? pound(s).(Type an integer or a decimal rounded to one decimal place asneeded.)b. Find the median.The median is ? pound(s).(Type an integer or a decimal rounded to one decimal place asneeded.)c. Find the mode.Select the correct choice below and, if necessary, fill in the answer box to complete your choice.A. The mode(s) is(are) ? pound(s).(Type an integer or a decimal. Do not round. Use a comma to separate answers as needed.)B. There is no mode.d. Find the midrange.The midrange is ? pound(s).(Type an integer or a decimal rounded to one decimal place asneeded.)e. Are the results likely to be representative of all players in that sport's league?A. The results are not likely to be representative because the median is not equal to the mode.B. The results are likely to be representative because a championship team is most likely representative of the entire league.C. The results are not likely to be representative because the median is not equal to the mean.D. The results are not likely to be representative because the championship team may not be representative of the entire league. Vu is three times as old as Wu. In 25 years Wu will be twice as old as Vu. How old is Vu now? What is the minimum thickness of coating which should be placed on a lens in order to minimize reflection of 566 nm light? The index of refraction of the coating material is 1.46 and the index of the glass is 1.71. Suppose you have read two different books on world war 2 and each book has different arguments about how the war started which of the following sources provides the best support for the authors arguments a second degree equation in one variable example how many solutions does it have ?a second degree equation in one variable example how many solutions does it have ? is it possible to have many solutions or no solutions tions give an example for each A bill passes the general assembly, but the governor vetoes it. Its chances or becoming law are now slim. What is the most likely reason for the bill's slim chances? A. The bill must receive two-thirds of the vote in the house of delegates and a majority in the Senate to override the veto.B. The bill must receive a certain number of signatures from the voters of Maryland to override the veto.C. The bill must receive three-fifths of the vote in both chambers of the general assembly to override the veto.D. The bill must receive unanimous support by all the seven judges on the court of appeals to override the veto. Please answer my question Please answer this question now in the derivation of the time period of a pendulum in electric field when considering the fbd of bob to find the g effective why do we neglect tension